電子書籍の厳選無料作品が豊富!

物理の問題なんです・・。
電荷qが原点付近にあり、2つのQの電荷がx軸上のx=aおよびx=-aに固定されている(q>0、Q>0)。qがx軸上に沿って運動するとき、原点からの変位xがaに比べて微小であるとしてqに働く力を求めよ。
という問題なんですが・・クーロンの法則をそのまま使うやり方ではなく、ベクトルの電場Eを使って求めるやり方が分かる人はいますでしょうか。。教えていただきたいです(>_<)

A 回答 (2件)

こんにちは。



私が回答したこれのことですか。
http://oshiete.goo.ne.jp/qa/6273364.html

x=a に固定されている電荷Qによって、座標(x、y、z)にある電荷qに働く力の大きさ(絶対値)は、
F = kQq/距離の2乗
 = kQq/{(x-a)^2+(y-0)^2+(z-0)^2}
 = kQq/{(x-a)^2+y^2+z^2}

その力をベクトルで現せば、
F→ = F × Qとqを結ぶ直線と平行な単位ベクトル
 = kQq/{(x-a)^2+y^2+z^2} × (x-a,y-0,z-0)/|(x-a,y-0,z-0)|
 = kQq/{(x-a)^2+y^2+z^2}^(3/2) × (x-a,y,z)

F→をqで割れば電場です。これをE1→と名づけます。
E1→ = kQ/|(x-a)^2+y^2+z^2|(3/2) × (x-a,y,z)

同様に、x=-a の電荷による電場E2→は、
E2→ = kQ/|(x+a)^2+y^2+z^2|^(3/2) × (x+a,y,z)

E1→とE2→を重ね合わせると、
Etot→ = E1→ + E2→
 = kQ/|(x-a)^2+y^2+z^2|^(3/2) × (x-a,y,z) + kQ/|(x+a)^2+y^2+z^2|^(3/2) × (x+a,y,z)

Etot→ のx成分Ex、y成分Ey、z成分Ezは、

Ex = kQ(x-a)/|(x-a)^2+y^2+z^2|^(3/2) + kQ(x+a)/|(x+a)^2+y^2+z^2|^(3/2)

Ey = kQy/|(x-a)^2+y^2+z^2|^(3/2) + kQy/|(x+a)^2+y^2+z^2|^(3/2)

Ez = kQz/|(x-a)^2+y^2+z^2|^(3/2) + kQz/|(x+a)^2+y^2+z^2|^(3/2)

以上のことは、
Etot→ = Exi→ + Eyj→ + Ezk→
とも表せることは、もちろんのことです。


「x軸上」であれば、y=z=0なので、

Ex = kQ(x-a)/|(x-a)^2+0^2+0^2|^(3/2) + kQ(x+a)/|(x+a)^2+0^2+0^2|^(3/2)
 = kQ(x-a)/|x-a|^3 + kQ(x+a)/|x+a|^3
 = kQ(x-a)/{(x-a)^2・|x-a|} + kQ(x+a)/{(x+a)^2・|x+a|}
 = kQ/{(x-a)・|x-a|} + kQ/{(x+a)・|x+a|}

Ey = 0

Ez = 0

-a<x<a のとき
Ex = kQ/{(x-a)・(-(x-a))} + kQ/{(x+a)・(x+a)}
 = -kQ/(x-a)^2 + kQ/(x+a)^2
 = kQ{-(x+a)^2 + (x-a)^2}/{(x-a)^2(x+a)^2}
 = -4kQax/(x^2-a^2)^2

原点からの変位xがaに比べて微小ということは、x^2≪a^2 なので、
Ex ≒ -4kQax/(0^2-a^2)^2
 = -4kQx/a^3

以上のことから、
Etot→ = (-4kQx/a^3, 0, 0)

qに働く力は、
qEtot→ = (-4kQqx/a^3, 0, 0)
    • good
    • 0
この回答へのお礼

ありがとうございます。助かりました

お礼日時:2010/11/07 17:09

a > 0 としても一般性は失われない。

x = - a にある電荷 Q が座標 x の点に作る電場ベクトルを E(x) とすると、x = a にある電荷 Q が |x| < a なる x に作る電場ベクトルは - E(-x)。それらふたつの電荷の作る合成電場ベクトルは
Et(x) = E(x) - E(-x) 。
|x| << a のときは
E(x) = E(0) + x dE/dx(0)
としてよいので
Et(x) = {E(0) + x dE/dx(0)} - {E(0) + (-x)dE/dx(0)}
   = 2 x dE/dx(0) 。
電荷 q に働く力は
F = q Et(x)
 = 2 q x dE/dx(0) 。

具体的には
E(x) = k Q / (x + a)^2
なので
dE/dx = - 2 k Q / (x + a)^3 、
dE/dx(0) = - 2 k Q / a^3 。
よって
F = - 4 k q Q x / a^3 。
    • good
    • 0
この回答へのお礼

ありがとうございました。分かりやすかったです。

お礼日時:2010/11/07 17:10

お探しのQ&Aが見つからない時は、教えて!gooで質問しましょう!